1
$\begingroup$

Possible Duplicate:
Lebesgue integral question concerning orders of limit and integration

I happened upon the following integration exercise. I don't know how standard it is, but it caught my eye, and I am wondering how best to approach it.

Suppose that $\phi \in L_1(\mu)$ is nonnegative, with $\alpha \in \mathbb{R}$ such that $\alpha>0$. Compute the value of the following:

$\lim_{n \to \infty} n \cdot \int \log (1 + (\phi(x)/n)^\alpha) d\mu (x).$

The author gives a hint: separately treat the cases $\alpha < 1$, $\alpha = 1$, $\alpha > 1$. I am wondering if such case-work will demonstrate divergence in at least one case. That aside, this is the first time I came across an Lebesgue integration exercise with a logarithm present, and it is throwing me! If anyone visiting the site today has performed a computation at this level, and would be up for an assist, I could use the help and would appreciate it! Thanks.

  • 1
    http://math.stackexchange.com/questions/81800/lebesgue-integral-question-concerning-orders-of-limit-and-integration/85386#85386 may interest you.2012-03-02

0 Answers 0